Question

Economist: On average, the emergency treatment for an elderly person for injuries resulting from a fall costs $11,000. A new therapeutic program can significantly reduce an elderly person’s chances of falling. Though obviously desirable for many reasons, this treatment program will cost $12,500 and thus cannot be justified.

Which of the following, if true, most seriously undermines the conclusion of the argument?

 

(This question is from Official Guide. Therefore, because of copyrights, the complete question cannot be copied here. The question can be accessed at GMAT Club)

Solution

Understanding the passage.

Economist: On average, the emergency treatment for an elderly person for injuries resulting from a fall costs $11,000.

On average,

When an elderly person falls and gets injured, the emergency treatment of such injuries costs $11,000.

A new therapeutic program can significantly reduce an elderly person’s chances of falling. 

A new therapeutic program -> Significant reduction in an elderly person’s chances of falling

Though obviously desirable for many reasons, this treatment program will cost $12,500 and thus cannot be justified.

The treatment program is clearly good for many reasons. However, since the program will cost $12,500 (i.e., more than the costs associated with emergency treatment of injuries resulting from a fall), the program cannot be justified.

The GIST.

Conclusion: The therapeutic program cannot be justified.

Support: It Costs $12,500 i.e., more than the average cost of the emergency treatment of injuries resulting from a fall.

The GAPS

How can we justify the therapeutic program even though it costs more than the emergency treatment of injuries resulting from a fall?

  1. Perhaps, the program, on average, saves more than one fall per elderly person (an elderly person can fall more than once in his lifetime)
  2. Perhaps, the elderly person loses income when he undergoes treatment for his injuries. Thus, the economic impact of a fall is more than just the cost of the treatment.
  3. Maybe people are willing to pay more than the cost of the treatment for a program that helps them not undergo the whole ordeal of pain, hospitalization, treatment, and medication in the first place.

There can, of course, be more gaps.

The Evaluation

(A) Incorrect.

This option has NO IMPACT on the argument.

The option compares the number of serious falls between two sets of elderly people:

  1. those who had followed the program for only a few months (i.e., those who did not follow the program for its minimum recommended duration)
  2. those who had followed the program for its recommended minimum duration of one year

The option says that the first group had more serious falls than the second group. Thus, the option suggests that the program is effective in bringing down the number of falls.

However, the argument already takes into account the idea that the program is effective in reducing the chances of a fall. The argument says that despite its effectiveness, since it costs more than the treatment of a fall, the program is not justified. This reasoning is nowhere impacted by this option.

(B) Incorrect

This option has NO IMPACT on the argument.

The option compares the frequency of falls resulting in serious injuries between two sets of elderly people:

  1. Those living in nursing homes
  2. Those living alone at home

The option says that the frequency of such falls is smaller in the first set of people than in the second set of people.

This option indicates another way to reduce the chances of a fall – by admitting elderly people living alone to a nursing home.

However, this idea has no relevance to the argument at hand.

(C) Correct

The option says that:

  1. Injuries resulting from a fall frequently result in long-term pain
  2. The medication for this pain is not counted in the emergency treatment costs talked about above

The option talks about an aspect of cost missed in the argument. If we take this cost into consideration, it is perhaps possible that the total medical cost may go beyond $12,500. Thus, the option indicates that the therapeutic program can be justified.

Moreover, apart from the financial aspect, long-term pain sounds like a very undesirable thing. So, perhaps people would be willing to pay for a program that helps prevent that – despite the seeming high cost.

So, this option weakens the argument.

(D) Incorrect

This option has NO IMPACT for almost the same reason as option A had no impact.

This option says that the therapeutic program avoids something (medication) that can increase the likelihood of a serious fall in an elderly person.

Thus, this option supports that the program reduces the chances of a fall of an elderly person.

However, the argument already takes into account the idea that the program is effective in reducing the chances of a fall. The argument says that despite its effectiveness, since it costs more than the treatment of a fall, the program is not justified. This reasoning is nowhere impacted by this option.

(E) Incorrect

This option has NO IMPACT on the argument.

The option says that a significant component of the costs of the program is something (regular visits by health care professionals) whose costs increase more rapidly than the costs of other elements of the program.

Does the option indicate that the costs of this program will increase more rapidly than the emergency treatment costs?

NOT AT ALL.

This is a mistake some of you may be making.

The option just compares two components of the costs of the program; it doesn’t compare the growth of the costs of the program with the growth of the costs of the treatment. Even if the significant component of the program grows in costs faster than the other component, the overall costs of the program can still grow significantly slower than the emergency treatment costs.

Leave a comment

Leave a Reply

Share this:

Like this:

Like Loading...

Discover more from GMAT with CJ

Subscribe now to keep reading and get access to the full archive.

Continue reading